What is the Correct Approach to Solving This 3D Fourier Transform Problem?

Click For Summary
The discussion centers on solving a 3D Fourier transform problem involving the function f(r) such that the integral of f(r) multiplied by an exponential term equals 1/w². The user initially attempted to compute the Fourier transform of 1/w² using residue calculus but encountered complex exponential terms and roots that complicated the solution. They seek guidance on whether to utilize polar coordinates for this problem and are interested in general methods for n-dimensional Fourier transformations. A suggestion is made to try polar coordinates as a potential approach. The conversation highlights the challenges of integrating in higher dimensions and the need for effective transformation techniques.
Sina
Messages
119
Reaction score
0

Homework Statement


What is the function f(r) s.t

int {d<sup>3</sup>r.f(r).e<sup>-iw.r</sup>= 1/w<sup>2</sup>}

where w = (kx,ky,kz)

Homework Equations


None

The Attempt at a Solution


I tried to directly take Fourier transform of 1/w2 as \int{ d<sup>3</sup>r.1/w<sup>2</sup>.e<sup>iw.r</sup>}. I started integrating dkx bu calculus of residues, calling the denominator kx2 + c2 and evaluating residues at kx = ic with a semi circle in the lower plane etc. However the integral I get from here is with roots and strange exponential terms so I stopped here. So I am asking for a line of approach should I be working in polar coordinates to solve this questions. And also what are some general methods approach n-dimensional Fourier transformations.Thanks
 
Last edited:
Physics news on Phys.org
Try polar coordinates instead. I haven't worked it out, but that's what I'd try.
 
Question: A clock's minute hand has length 4 and its hour hand has length 3. What is the distance between the tips at the moment when it is increasing most rapidly?(Putnam Exam Question) Answer: Making assumption that both the hands moves at constant angular velocities, the answer is ## \sqrt{7} .## But don't you think this assumption is somewhat doubtful and wrong?

Similar threads

Replies
5
Views
2K
Replies
8
Views
2K
  • · Replies 6 ·
Replies
6
Views
2K
  • · Replies 6 ·
Replies
6
Views
3K
  • · Replies 1 ·
Replies
1
Views
1K
Replies
1
Views
2K
  • · Replies 5 ·
Replies
5
Views
2K
  • · Replies 9 ·
Replies
9
Views
2K
  • · Replies 7 ·
Replies
7
Views
3K
  • · Replies 1 ·
Replies
1
Views
1K